Akademisyenler öncülüğünde matematik/fizik/bilgisayar bilimleri soru cevap platformu
0 beğenilme 0 beğenilmeme
293 kez görüntülendi
$\sum_{n=1}^\infty \frac{\sin n}n$ serisi mutlak yakınsak mıdır?

(Burada $n$ radyan)
Lisans Matematik kategorisinde (6.1k puan) tarafından  | 293 kez görüntülendi

Degildir. Cunku harmonik seri iraksar ? 

 

Yanlis dusundum sanirim  ama iraksadigina emin gibiyim

Aslında ilginç bir şey değil. Biraz analiz tekniği kullanmak yeterli.
Ya icime yatmiyor ama soyle birsey deneyecegim

 

$100\frac{|\sin n|}{n} $ dizisinin elemanlari $\frac{1}{n}$ dizisindeki elemanlarindan buyuk. Harmonik seri iraksadigina gore, $\sum100\frac{|\sin n|}{n} $ da iraksar. Bastaki katsayi olan $100$ den de kurtulabiliriz sanirim
poff bu da yanlis cozum oldu dusunup cevaplamak lazim tahmin etmek yerine. Nedense harmonik seri ile kiyaslamak istiyorum hep
Harmonik seri ile kıyaslamak doğru fikir, ama bu serinin her terimini değil, aradan "büyük" olanları seçip yapabiliriz (ben öyle yaptım).

1 cevap

1 beğenilme 0 beğenilmeme
Önce şunu gösterelim:
    Her $n\in\mathbb{Z}$ için $|\sin n|,|\sin(n+1)|,|\sin(n+2)|$ sayılarından en az biri  $\frac12$ den büyük  olur.
    ($ \mathbb{R} $ de) $(n-1,n+3)$ aralığında (uzunluğu$=4>\pi$ olduğu için) en az bir $ \frac{2k+1}{2}\pi (k\in\mathbb{Z})$ sayısı vardır.
    ($ \frac\pi3<2 $ olduğu için) $\frac{2k+1}{2}\pi\pm\frac\pi3  $ sayılarından en az biri  $(n-1,n+3)$ aralığındadır.
    ($ \frac\pi3>1 $ olduğu için)
    $\frac{2k+1}{2}\pi-\frac\pi3 \in (n-1,n+3) $ ise $m\in(\frac{2k+1}{2}\pi-\frac\pi3,\frac{2k+1}{2}\pi)  $ olacak şekilde;
    $\frac{2k+1}{2}\pi+\frac\pi3 \in (n-1,n+3) $ ise $m\in(\frac{2k+1}{2}\pi,\frac{2k+1}{2}\pi+\frac\pi3)  $ olacak şekilde
    bir $ m\in\mathbb{Z} $ vardır.

$(\frac{2k+1}{2}\pi-\frac\pi3,\frac{2k+1}{2}\pi+\frac\pi3)  $ aralığında $|\sin x|>\frac12$ olduğundan)
    Her iki durumda da, $n-1<m<n+3$ ve $ |\sin m|>\frac12 $ olur. İddiamız kanıtlanmıştır.
    
    Şimdi serimize dönelim:
    
    Yukarıdaki önermeden, (her $ k\geq1 $ için):    
    $ \frac{|\sin(3k-2)|}{3k-2}+\frac{|\sin(3k-1)|}{3k-1}+\frac{|\sin(3k)|}{3k}\geq\frac1{3k}\left(|\sin(3k-2)|+|\sin(3k-1)|+|\sin(3k)|\right)>\frac1{3k}\frac12=\frac16\frac1{k} $

elde edilir. Bu eşitsizlikler ($k=1,2,\ldots,n$ için) yazılıp, taraf tarafa toplandığında:    
     $ \frac{|\sin1|}{1}+\frac{|\sin2|}{2}+\cdots+\frac{|\sin(3n)|}{3n}>\frac16(1+\frac12+\cdots+\frac1{n} ) $ bulunur.
     $1+\frac12+\cdots+\frac1{n}   $, harmonik serinin $ n $ nci kısmi toplamı olup, (pozitif terimli) harmonik serinin ıraksak oluşundan, $(1+\frac12+\cdots+\frac1{n} )_{n=1}^\infty  $ dizisi sınırsızdır.
     Yularıdaki eşitsizlikten dolayı, $ \sum_{n=1}^{\infty}\frac{|\sin n|}{n} $ serisinin kısmi toplamlar dizisi de sınırsız, dolayısıyla kısmi toplamlar dizisi ıraksak olur.

Bu da, $\sum_{n=1}^{\infty}\frac{|\sin n|}{n}  $ serisinin ıraksak olduğunu gösterir.
(6.1k puan) tarafından 
tarafından düzenlendi
20,210 soru
21,736 cevap
73,302 yorum
1,909,118 kullanıcı